Show that $limlimits_{x to infty}frac{ln x}{x}=0$ from definition.











up vote
1
down vote

favorite












Show that $displaystylelim_{x to infty}frac{ln x}{x}=0$.



I know this is a simple application of the L'Hopital's rule, but can we also show this from the $displaystyleepsilon-delta$ definition?



I am stuck because while it is easy to find a lower bound for the denominator $x$, the numerator does not have an upper bound - it merely increases less fast than the denominator. Is there a way to manipulate the expression to get a bound?










share|cite|improve this question
























  • $ln(x)leq x$ for $x>0$ and show it for the function $frac{1}{x}$.
    – Yadati Kiran
    Nov 15 at 18:38












  • @Rustyn there is one for infinite limits
    – gt6989b
    Nov 15 at 18:38










  • @gt6989b many thanks, I'll have a look
    – Rustyn
    Nov 15 at 18:39






  • 3




    If you let $x = e^y$, then it's equivalent to show that $$ lim_{y to infty} frac{y}{e^y} = 0 $$
    – Omnomnomnom
    Nov 15 at 18:42






  • 1




    Well its just an immediate but non-obvious consequence of the inequality $log xleq x-1$ or if you prefer the simpler inequality $log x<x$. The non-obvious part is related to the fact that we need to replace $x$ with $sqrt{x} $ in above inequality to get $log x<2sqrt{x}$.
    – Paramanand Singh
    Nov 15 at 18:53















up vote
1
down vote

favorite












Show that $displaystylelim_{x to infty}frac{ln x}{x}=0$.



I know this is a simple application of the L'Hopital's rule, but can we also show this from the $displaystyleepsilon-delta$ definition?



I am stuck because while it is easy to find a lower bound for the denominator $x$, the numerator does not have an upper bound - it merely increases less fast than the denominator. Is there a way to manipulate the expression to get a bound?










share|cite|improve this question
























  • $ln(x)leq x$ for $x>0$ and show it for the function $frac{1}{x}$.
    – Yadati Kiran
    Nov 15 at 18:38












  • @Rustyn there is one for infinite limits
    – gt6989b
    Nov 15 at 18:38










  • @gt6989b many thanks, I'll have a look
    – Rustyn
    Nov 15 at 18:39






  • 3




    If you let $x = e^y$, then it's equivalent to show that $$ lim_{y to infty} frac{y}{e^y} = 0 $$
    – Omnomnomnom
    Nov 15 at 18:42






  • 1




    Well its just an immediate but non-obvious consequence of the inequality $log xleq x-1$ or if you prefer the simpler inequality $log x<x$. The non-obvious part is related to the fact that we need to replace $x$ with $sqrt{x} $ in above inequality to get $log x<2sqrt{x}$.
    – Paramanand Singh
    Nov 15 at 18:53













up vote
1
down vote

favorite









up vote
1
down vote

favorite











Show that $displaystylelim_{x to infty}frac{ln x}{x}=0$.



I know this is a simple application of the L'Hopital's rule, but can we also show this from the $displaystyleepsilon-delta$ definition?



I am stuck because while it is easy to find a lower bound for the denominator $x$, the numerator does not have an upper bound - it merely increases less fast than the denominator. Is there a way to manipulate the expression to get a bound?










share|cite|improve this question















Show that $displaystylelim_{x to infty}frac{ln x}{x}=0$.



I know this is a simple application of the L'Hopital's rule, but can we also show this from the $displaystyleepsilon-delta$ definition?



I am stuck because while it is easy to find a lower bound for the denominator $x$, the numerator does not have an upper bound - it merely increases less fast than the denominator. Is there a way to manipulate the expression to get a bound?







calculus real-analysis limits limits-without-lhopital






share|cite|improve this question















share|cite|improve this question













share|cite|improve this question




share|cite|improve this question








edited Nov 16 at 11:55









Martin Sleziak

44.4k7115268




44.4k7115268










asked Nov 15 at 18:33









Bunbury

627




627












  • $ln(x)leq x$ for $x>0$ and show it for the function $frac{1}{x}$.
    – Yadati Kiran
    Nov 15 at 18:38












  • @Rustyn there is one for infinite limits
    – gt6989b
    Nov 15 at 18:38










  • @gt6989b many thanks, I'll have a look
    – Rustyn
    Nov 15 at 18:39






  • 3




    If you let $x = e^y$, then it's equivalent to show that $$ lim_{y to infty} frac{y}{e^y} = 0 $$
    – Omnomnomnom
    Nov 15 at 18:42






  • 1




    Well its just an immediate but non-obvious consequence of the inequality $log xleq x-1$ or if you prefer the simpler inequality $log x<x$. The non-obvious part is related to the fact that we need to replace $x$ with $sqrt{x} $ in above inequality to get $log x<2sqrt{x}$.
    – Paramanand Singh
    Nov 15 at 18:53


















  • $ln(x)leq x$ for $x>0$ and show it for the function $frac{1}{x}$.
    – Yadati Kiran
    Nov 15 at 18:38












  • @Rustyn there is one for infinite limits
    – gt6989b
    Nov 15 at 18:38










  • @gt6989b many thanks, I'll have a look
    – Rustyn
    Nov 15 at 18:39






  • 3




    If you let $x = e^y$, then it's equivalent to show that $$ lim_{y to infty} frac{y}{e^y} = 0 $$
    – Omnomnomnom
    Nov 15 at 18:42






  • 1




    Well its just an immediate but non-obvious consequence of the inequality $log xleq x-1$ or if you prefer the simpler inequality $log x<x$. The non-obvious part is related to the fact that we need to replace $x$ with $sqrt{x} $ in above inequality to get $log x<2sqrt{x}$.
    – Paramanand Singh
    Nov 15 at 18:53
















$ln(x)leq x$ for $x>0$ and show it for the function $frac{1}{x}$.
– Yadati Kiran
Nov 15 at 18:38






$ln(x)leq x$ for $x>0$ and show it for the function $frac{1}{x}$.
– Yadati Kiran
Nov 15 at 18:38














@Rustyn there is one for infinite limits
– gt6989b
Nov 15 at 18:38




@Rustyn there is one for infinite limits
– gt6989b
Nov 15 at 18:38












@gt6989b many thanks, I'll have a look
– Rustyn
Nov 15 at 18:39




@gt6989b many thanks, I'll have a look
– Rustyn
Nov 15 at 18:39




3




3




If you let $x = e^y$, then it's equivalent to show that $$ lim_{y to infty} frac{y}{e^y} = 0 $$
– Omnomnomnom
Nov 15 at 18:42




If you let $x = e^y$, then it's equivalent to show that $$ lim_{y to infty} frac{y}{e^y} = 0 $$
– Omnomnomnom
Nov 15 at 18:42




1




1




Well its just an immediate but non-obvious consequence of the inequality $log xleq x-1$ or if you prefer the simpler inequality $log x<x$. The non-obvious part is related to the fact that we need to replace $x$ with $sqrt{x} $ in above inequality to get $log x<2sqrt{x}$.
– Paramanand Singh
Nov 15 at 18:53




Well its just an immediate but non-obvious consequence of the inequality $log xleq x-1$ or if you prefer the simpler inequality $log x<x$. The non-obvious part is related to the fact that we need to replace $x$ with $sqrt{x} $ in above inequality to get $log x<2sqrt{x}$.
– Paramanand Singh
Nov 15 at 18:53










2 Answers
2






active

oldest

votes

















up vote
6
down vote



accepted










Assuming you define the $ln(cdot) $ by the integral, then it is more or less enough to notice that
$$0leq int^x_1frac 1 t, dtleq int^x_1frac 1{ sqrt t} , dtleq int^x_0frac{1}{sqrt t} , dt$$
This leads to
$$left |frac{ln(x)} {x} right|leq left|frac{2sqrt{x}}{x} right|$$
The rest is straightforward.






share|cite|improve this answer























  • Ah yes! I forgot that's how we defined log function. Thank you!
    – Bunbury
    Nov 15 at 18:51










  • @Bunbury glad I could help, I made it a little bit easier now.
    – Shashi
    Nov 15 at 18:58


















up vote
1
down vote













For all $x > 0, ln x < sqrt x$



$frac {ln x}{x} < frac {1}{sqrt x}$



When $N > frac {1}{epsilon^2},$ then $x>N implies frac {ln x}{x} < frac {1}{sqrt x} < epsilon$






share|cite|improve this answer





















    Your Answer





    StackExchange.ifUsing("editor", function () {
    return StackExchange.using("mathjaxEditing", function () {
    StackExchange.MarkdownEditor.creationCallbacks.add(function (editor, postfix) {
    StackExchange.mathjaxEditing.prepareWmdForMathJax(editor, postfix, [["$", "$"], ["\\(","\\)"]]);
    });
    });
    }, "mathjax-editing");

    StackExchange.ready(function() {
    var channelOptions = {
    tags: "".split(" "),
    id: "69"
    };
    initTagRenderer("".split(" "), "".split(" "), channelOptions);

    StackExchange.using("externalEditor", function() {
    // Have to fire editor after snippets, if snippets enabled
    if (StackExchange.settings.snippets.snippetsEnabled) {
    StackExchange.using("snippets", function() {
    createEditor();
    });
    }
    else {
    createEditor();
    }
    });

    function createEditor() {
    StackExchange.prepareEditor({
    heartbeatType: 'answer',
    convertImagesToLinks: true,
    noModals: true,
    showLowRepImageUploadWarning: true,
    reputationToPostImages: 10,
    bindNavPrevention: true,
    postfix: "",
    imageUploader: {
    brandingHtml: "Powered by u003ca class="icon-imgur-white" href="https://imgur.com/"u003eu003c/au003e",
    contentPolicyHtml: "User contributions licensed under u003ca href="https://creativecommons.org/licenses/by-sa/3.0/"u003ecc by-sa 3.0 with attribution requiredu003c/au003e u003ca href="https://stackoverflow.com/legal/content-policy"u003e(content policy)u003c/au003e",
    allowUrls: true
    },
    noCode: true, onDemand: true,
    discardSelector: ".discard-answer"
    ,immediatelyShowMarkdownHelp:true
    });


    }
    });














    draft saved

    draft discarded


















    StackExchange.ready(
    function () {
    StackExchange.openid.initPostLogin('.new-post-login', 'https%3a%2f%2fmath.stackexchange.com%2fquestions%2f3000091%2fshow-that-lim-limits-x-to-infty-frac-ln-xx-0-from-definition%23new-answer', 'question_page');
    }
    );

    Post as a guest















    Required, but never shown

























    2 Answers
    2






    active

    oldest

    votes








    2 Answers
    2






    active

    oldest

    votes









    active

    oldest

    votes






    active

    oldest

    votes








    up vote
    6
    down vote



    accepted










    Assuming you define the $ln(cdot) $ by the integral, then it is more or less enough to notice that
    $$0leq int^x_1frac 1 t, dtleq int^x_1frac 1{ sqrt t} , dtleq int^x_0frac{1}{sqrt t} , dt$$
    This leads to
    $$left |frac{ln(x)} {x} right|leq left|frac{2sqrt{x}}{x} right|$$
    The rest is straightforward.






    share|cite|improve this answer























    • Ah yes! I forgot that's how we defined log function. Thank you!
      – Bunbury
      Nov 15 at 18:51










    • @Bunbury glad I could help, I made it a little bit easier now.
      – Shashi
      Nov 15 at 18:58















    up vote
    6
    down vote



    accepted










    Assuming you define the $ln(cdot) $ by the integral, then it is more or less enough to notice that
    $$0leq int^x_1frac 1 t, dtleq int^x_1frac 1{ sqrt t} , dtleq int^x_0frac{1}{sqrt t} , dt$$
    This leads to
    $$left |frac{ln(x)} {x} right|leq left|frac{2sqrt{x}}{x} right|$$
    The rest is straightforward.






    share|cite|improve this answer























    • Ah yes! I forgot that's how we defined log function. Thank you!
      – Bunbury
      Nov 15 at 18:51










    • @Bunbury glad I could help, I made it a little bit easier now.
      – Shashi
      Nov 15 at 18:58













    up vote
    6
    down vote



    accepted







    up vote
    6
    down vote



    accepted






    Assuming you define the $ln(cdot) $ by the integral, then it is more or less enough to notice that
    $$0leq int^x_1frac 1 t, dtleq int^x_1frac 1{ sqrt t} , dtleq int^x_0frac{1}{sqrt t} , dt$$
    This leads to
    $$left |frac{ln(x)} {x} right|leq left|frac{2sqrt{x}}{x} right|$$
    The rest is straightforward.






    share|cite|improve this answer














    Assuming you define the $ln(cdot) $ by the integral, then it is more or less enough to notice that
    $$0leq int^x_1frac 1 t, dtleq int^x_1frac 1{ sqrt t} , dtleq int^x_0frac{1}{sqrt t} , dt$$
    This leads to
    $$left |frac{ln(x)} {x} right|leq left|frac{2sqrt{x}}{x} right|$$
    The rest is straightforward.







    share|cite|improve this answer














    share|cite|improve this answer



    share|cite|improve this answer








    edited Nov 15 at 18:56

























    answered Nov 15 at 18:39









    Shashi

    6,9191527




    6,9191527












    • Ah yes! I forgot that's how we defined log function. Thank you!
      – Bunbury
      Nov 15 at 18:51










    • @Bunbury glad I could help, I made it a little bit easier now.
      – Shashi
      Nov 15 at 18:58


















    • Ah yes! I forgot that's how we defined log function. Thank you!
      – Bunbury
      Nov 15 at 18:51










    • @Bunbury glad I could help, I made it a little bit easier now.
      – Shashi
      Nov 15 at 18:58
















    Ah yes! I forgot that's how we defined log function. Thank you!
    – Bunbury
    Nov 15 at 18:51




    Ah yes! I forgot that's how we defined log function. Thank you!
    – Bunbury
    Nov 15 at 18:51












    @Bunbury glad I could help, I made it a little bit easier now.
    – Shashi
    Nov 15 at 18:58




    @Bunbury glad I could help, I made it a little bit easier now.
    – Shashi
    Nov 15 at 18:58










    up vote
    1
    down vote













    For all $x > 0, ln x < sqrt x$



    $frac {ln x}{x} < frac {1}{sqrt x}$



    When $N > frac {1}{epsilon^2},$ then $x>N implies frac {ln x}{x} < frac {1}{sqrt x} < epsilon$






    share|cite|improve this answer

























      up vote
      1
      down vote













      For all $x > 0, ln x < sqrt x$



      $frac {ln x}{x} < frac {1}{sqrt x}$



      When $N > frac {1}{epsilon^2},$ then $x>N implies frac {ln x}{x} < frac {1}{sqrt x} < epsilon$






      share|cite|improve this answer























        up vote
        1
        down vote










        up vote
        1
        down vote









        For all $x > 0, ln x < sqrt x$



        $frac {ln x}{x} < frac {1}{sqrt x}$



        When $N > frac {1}{epsilon^2},$ then $x>N implies frac {ln x}{x} < frac {1}{sqrt x} < epsilon$






        share|cite|improve this answer












        For all $x > 0, ln x < sqrt x$



        $frac {ln x}{x} < frac {1}{sqrt x}$



        When $N > frac {1}{epsilon^2},$ then $x>N implies frac {ln x}{x} < frac {1}{sqrt x} < epsilon$







        share|cite|improve this answer












        share|cite|improve this answer



        share|cite|improve this answer










        answered Nov 15 at 18:53









        Doug M

        43k31752




        43k31752






























            draft saved

            draft discarded




















































            Thanks for contributing an answer to Mathematics Stack Exchange!


            • Please be sure to answer the question. Provide details and share your research!

            But avoid



            • Asking for help, clarification, or responding to other answers.

            • Making statements based on opinion; back them up with references or personal experience.


            Use MathJax to format equations. MathJax reference.


            To learn more, see our tips on writing great answers.





            Some of your past answers have not been well-received, and you're in danger of being blocked from answering.


            Please pay close attention to the following guidance:


            • Please be sure to answer the question. Provide details and share your research!

            But avoid



            • Asking for help, clarification, or responding to other answers.

            • Making statements based on opinion; back them up with references or personal experience.


            To learn more, see our tips on writing great answers.




            draft saved


            draft discarded














            StackExchange.ready(
            function () {
            StackExchange.openid.initPostLogin('.new-post-login', 'https%3a%2f%2fmath.stackexchange.com%2fquestions%2f3000091%2fshow-that-lim-limits-x-to-infty-frac-ln-xx-0-from-definition%23new-answer', 'question_page');
            }
            );

            Post as a guest















            Required, but never shown





















































            Required, but never shown














            Required, but never shown












            Required, but never shown







            Required, but never shown

































            Required, but never shown














            Required, but never shown












            Required, but never shown







            Required, but never shown







            Popular posts from this blog

            Biblatex bibliography style without URLs when DOI exists (in Overleaf with Zotero bibliography)

            ComboBox Display Member on multiple fields

            Is it possible to collect Nectar points via Trainline?